Difference between revisions of "2000 AMC 8 Problems/Problem 21"

(Solution and see also box)
(4 intermediate revisions by 4 users not shown)
Line 6: Line 6:
  
 
==Solution==
 
==Solution==
 +
Divide it into <math>2</math> cases:
 +
 +
1) Keiko and Ephriam both get <math>0</math> heads:
 +
This means that they both roll all tails, so there is only <math>1</math> way for this to happen.
 +
 +
2) Keiko and Ephriam both get <math>1</math> head:
 +
For Keiko, there is only <math>1</math> way for this to happen because he is only flipping 1 penny, but for Ephriam, there are 2 ways since there are <math>2</math> choices for when he can flip the head. So, in total there are <math>2 \cdot 1 = 2</math> ways for this case.
 +
 +
Thus, in total there are <math>3</math> ways that work. Since there are <math>2</math> choices for each coin flip (Heads or Tails), there are <math>2^3 = 8</math> total ways of flipping 3 coins.
 +
 +
Thus, since all possible coin flips of 3 coins are equally likely, the probability is <math>\boxed{(B) \frac38}</math>.
 +
 +
~pi_is_3.14
 +
 +
==Solution 2==
  
 
Let <math>K(n)</math> be the probability that Keiko gets <math>n</math> heads, and let <math>E(n)</math> be the probability that Ephriam gets <math>n</math> heads.
 
Let <math>K(n)</math> be the probability that Keiko gets <math>n</math> heads, and let <math>E(n)</math> be the probability that Ephriam gets <math>n</math> heads.
Line 21: Line 36:
 
<math>E(2) = \frac{1}{2}\cdot\frac{1}{2} = \frac{1}{4}</math>
 
<math>E(2) = \frac{1}{2}\cdot\frac{1}{2} = \frac{1}{4}</math>
  
The probability that Keiko gets <math>0</math> heads and Ephriam gets <math>0</math> heads is <math>K(0)\cdot E(0)</math>.  Simiarly for <math>1</math> head and <math>2</math> heads.  Thus, we have:
+
The probability that Keiko gets <math>0</math> heads and Ephriam gets <math>0</math> heads is <math>K(0)\cdot E(0)</math>.  Similarly for <math>1</math> head and <math>2</math> heads.  Thus, we have:
  
 
<math>P = K(0)\cdot E(0) + K(1)\cdot E(1) + K(2)\cdot E(2)</math>
 
<math>P = K(0)\cdot E(0) + K(1)\cdot E(1) + K(2)\cdot E(2)</math>
Line 30: Line 45:
  
 
Thus the answer is <math>\boxed{B}</math>.
 
Thus the answer is <math>\boxed{B}</math>.
 +
 +
== Video Solution ==
 +
https://youtu.be/a_Tfeb_6dqE Soo, DRMS, NM
  
 
==See Also==
 
==See Also==
  
 
{{AMC8 box|year=2000|num-b=20|num-a=22}}
 
{{AMC8 box|year=2000|num-b=20|num-a=22}}
 +
{{MAA Notice}}

Revision as of 17:21, 8 October 2022

Problem

Keiko tosses one penny and Ephraim tosses two pennies. The probability that Ephraim gets the same number of heads that Keiko gets is

$\text{(A)}\ \frac{1}{4}\qquad\text{(B)}\ \frac{3}{8}\qquad\text{(C)}\ \frac{1}{2}\qquad\text{(D)}\ \frac{2}{3}\qquad\text{(E)}\ \frac{3}{4}$

Solution

Divide it into $2$ cases:

1) Keiko and Ephriam both get $0$ heads: This means that they both roll all tails, so there is only $1$ way for this to happen.

2) Keiko and Ephriam both get $1$ head: For Keiko, there is only $1$ way for this to happen because he is only flipping 1 penny, but for Ephriam, there are 2 ways since there are $2$ choices for when he can flip the head. So, in total there are $2 \cdot 1 = 2$ ways for this case.

Thus, in total there are $3$ ways that work. Since there are $2$ choices for each coin flip (Heads or Tails), there are $2^3 = 8$ total ways of flipping 3 coins.

Thus, since all possible coin flips of 3 coins are equally likely, the probability is $\boxed{(B) \frac38}$.

~pi_is_3.14

Solution 2

Let $K(n)$ be the probability that Keiko gets $n$ heads, and let $E(n)$ be the probability that Ephriam gets $n$ heads.

$K(0) = \frac{1}{2}$

$K(1) = \frac{1}{2}$

$K(2) = 0$ (Keiko only has one penny!)

$E(0) = \frac{1}{2}\cdot\frac{1}{2} = \frac{1}{4}$

$E(1) = \frac{1}{2}\cdot\frac{1}{2} + \frac{1}{2}\cdot\frac{1}{2} = 2\cdot\frac{1}{4} = \frac{1}{2}$ (because Ephraim can get HT or TH)

$E(2) = \frac{1}{2}\cdot\frac{1}{2} = \frac{1}{4}$

The probability that Keiko gets $0$ heads and Ephriam gets $0$ heads is $K(0)\cdot E(0)$. Similarly for $1$ head and $2$ heads. Thus, we have:

$P = K(0)\cdot E(0) + K(1)\cdot E(1) + K(2)\cdot E(2)$

$P = \frac{1}{2}\cdot\frac{1}{4} + \frac{1}{2}\cdot\frac{1}{2} + 0$

$P = \frac{3}{8}$

Thus the answer is $\boxed{B}$.

Video Solution

https://youtu.be/a_Tfeb_6dqE Soo, DRMS, NM

See Also

2000 AMC 8 (ProblemsAnswer KeyResources)
Preceded by
Problem 20
Followed by
Problem 22
1 2 3 4 5 6 7 8 9 10 11 12 13 14 15 16 17 18 19 20 21 22 23 24 25
All AJHSME/AMC 8 Problems and Solutions

The problems on this page are copyrighted by the Mathematical Association of America's American Mathematics Competitions. AMC logo.png